How can one obtain conserved charges from a symmetry transformation in QFT?

  • Thread starter center o bass
  • Start date
  • Tags
    Charge Qft
In summary, the Lagrangian density has the conserved charges Q_a which are given by Q = \int d^3x j^0 = \int d^3 x \dot \phi_a \epsilon_{abc} nb \phi_c.
  • #1
center o bass
560
2

Homework Statement


From the Lagrangian density

[tex] L = \frac{1}2 \partial_\mu \phi_a \partial^\mu \phi_a - \frac{1}2 \phi_a \phi_a,[/tex]

where a = 1,2,3 and the transformation

[tex] \phi_a \to \phi _a + \theta \epsilon_{abc} n_b \phi_c [/tex]

show that one gets the conserved charges

[tex]Q_a = \int d^3x \epsilon_{abc}\dot{\phi}_b \phi_c.[/tex]

Homework Equations


The transformation is a symmetry of the Lagrangian so by Noethers theorem
we got a conserved current which is given by

[tex]j^\mu = \frac{\partial L}{\partial(\partial_\mu \phi_a)} \delta \phi_a = \partial^\mu \phi_a \epsilon_{abc} n_b \phi_c[/tex]

The Attempt at a Solution


The obvious conserved charge is just

[tex] Q = \int d^3x j^0 = \int d^3 x \dot \phi_a \epsilon_{abc} nb \phi_c [/tex]
but this is not the 3 different charges in the expression for Q_a. There is no normal vector n in that expresion and the time differentiated field has got b-index instead of an a index.
How can one get from the conserved current to the expression for these charges?
[
 
Last edited:
Physics news on Phys.org
  • #2
Your expression for the conserved current doesn't look correct. In the middle term, you sum over a, but in the expression on the right, a is a free index.
 
  • #3
vela said:
Your expression for the conserved current doesn't look correct. In the middle term, you sum over a, but in the expression on the right, a is a free index.

I agree and I have corrected it now. That was just a typo. Do you have any suggestion on how to go from the corrected expression to the conserved charge?
 
  • #4
center o bass said:
[tex]Q_a = \int d^3x \epsilon_{abc}\dot{\phi}_a \phi_c.[/tex]

Is there a typo here for the subscript on the left side?
The obvious conserved charge is just

[tex] Q = \int d^3x j^0 = \int d^3 x \dot \phi_a \epsilon_{abc} nb \phi_c [/tex]
but this is not the 3 different charges in the expression for Q_a.

Is the Lagrangian invariant under the transformation for any choice of the vector nb? If so, you should be able to get three independent conserved charges by choosing three independent vectors for n. For example, what would the conserved charge be if you choose n1 = 1 and n2 = n3 = 0?
 
  • #5
TSny said:
Is there a typo here for the subscript on the left side?Is the Lagrangian invariant under the transformation for any choice of the vector nb? If so, you should be able to get three independent conserved charges by choosing three independent vectors for n. For example, what would the conserved charge be if you choose n1 = 1 and n2 = n3 = 0?

Yes there was yet another typo. I have corrected that one too now. That's true.
Hmm.. The conserved current satisfy

[tex] \partial_\mu (\partial^\mu \phi_a \epsilon_{abc} n_b \phi_c) = 0[/tex]

I agree with you that one should be able to get 3 independent conserved charges (and also currents), so what if i chose all components to be zero except the a'th component?
I.e. choose

[tex] n_b = \delta _{ba}.[/tex]

Would it then be correct of me to write

[tex] \partial_\mu (\partial^\mu \phi_a \epsilon_{abc} \delta_{ab} \phi_c) =\partial_\mu ( \epsilon_{abc} \partial^\mu \phi_b \phi_c)= 0[/tex]

so that i get

[tex](j^\mu)_a = \epsilon_{abc} \partial^\mu \phi_b \phi_c?[/tex]
 
  • #6
center o bass said:
Yes there was yet another typo. I have corrected that one too now. That's true.
Hmm.. The conserved current satisfy

[tex] \partial_\mu (\partial^\mu \phi_a \epsilon_{abc} n_b \phi_c) = 0[/tex]

I agree with you that one should be able to get 3 independent conserved charges (and also currents), so what if i chose all components to be zero except the a'th component?
I.e. choose

[tex] n_b = \delta _{ba}.[/tex]

You don't want to use a subscript "a" here, since you are already using that symbol as a dummy summation index in jμ.

Would it then be correct of me to write

[tex] \partial_\mu (\partial^\mu \phi_a \epsilon_{abc} \delta_{ab} \phi_c) =\partial_\mu ( \epsilon_{abc} \partial^\mu \phi_b \phi_c)= 0[/tex]

See, the subscript "a" is occurring three times here on the left. That's not good.

You might try again by letting [tex] n_b = \delta _{bd}.[/tex] so that you're letting nd be the nonzero component of n.

You can actually go to your result for Q in your original post: [tex] Q = \int d^3x j^0 = \int d^3 x \dot \phi_a \epsilon_{abc} nb \phi_c [/tex] and make this substitution rather than starting way back at the expression for jμ.
 
Last edited:

1. What is a conserved charge in quantum field theory?

In quantum field theory (QFT), a conserved charge is a physical quantity that remains constant over time and is associated with a symmetry of the system. In other words, it is a property of a system that is conserved or does not change as the system evolves.

2. What are some examples of conserved charges in QFT?

Some examples of conserved charges in QFT include electric charge, baryon number, and lepton number. These charges are associated with symmetries of the electromagnetic, strong, and weak interactions, respectively.

3. How are conserved charges related to conservation laws in QFT?

Conserved charges are related to conservation laws in QFT through Noether's theorem, which states that for every continuous symmetry of a system, there exists a corresponding conserved quantity. This means that the conservation of a conserved charge is a consequence of a fundamental symmetry of the system.

4. Can conserved charges be changed or violated?

In general, conserved charges cannot be changed or violated. This is because they are associated with fundamental symmetries of the system, which are believed to be exact. However, there are some cases where conserved charges may appear to be violated due to quantum effects or external influences, but these violations are typically very small and can be accounted for in calculations.

5. How are conserved charges measured in experiments?

Conserved charges can be measured in experiments through various techniques, such as particle colliders or precision measurements of physical quantities. For example, electric charge can be measured by observing the deflection of charged particles in a magnetic field, while baryon and lepton numbers can be inferred from the decay products of particles. These measurements are crucial for testing the predictions of QFT and understanding the fundamental symmetries of the universe.

Similar threads

  • Advanced Physics Homework Help
Replies
1
Views
1K
  • Advanced Physics Homework Help
Replies
30
Views
5K
  • Advanced Physics Homework Help
Replies
1
Views
784
Replies
27
Views
2K
  • Advanced Physics Homework Help
2
Replies
57
Views
16K
  • Calculus and Beyond Homework Help
Replies
15
Views
2K
  • Advanced Physics Homework Help
Replies
1
Views
655
  • Advanced Physics Homework Help
Replies
17
Views
2K
  • Classical Physics
Replies
3
Views
2K
  • High Energy, Nuclear, Particle Physics
Replies
4
Views
1K
Back
Top